Đến nội dung

Hình ảnh

Inequalities From 2016 Mathematical Olympiads

* * * * * 3 Bình chọn

  • Please log in to reply
Chủ đề này có 139 trả lời

#61
Nguyenhuyen_AG

Nguyenhuyen_AG

    Trung úy

  • Thành viên nổi bật 2016
  • 945 Bài viết

Bài 29 (EMMO). Tìm số thực dương $\lambda$ nhỏ nhất sao cho bất đẳng thức
$$\frac{\sqrt[3]{a^3+b^3}}{a^2+b^2}+\frac{\sqrt[3]{b^3+c^3}}{b^2+c^2}+\frac{\sqrt[3]{c^3+a^3}}{c^2+a^2}\le \lambda\left(1+\frac{1}{ab+bc+ca}\right),$$

luôn đúng với mọi số dương $a,b,c$ thỏa mãn điều kiện $a+b+c=1.$


Nguyen Van Huyen
Ho Chi Minh City University Of Transport

#62
Nguyenhuyen_AG

Nguyenhuyen_AG

    Trung úy

  • Thành viên nổi bật 2016
  • 945 Bài viết

Bài 30 (EMMO). Cho ba số thực $a,b,c$ thỏa mãn điều kiện $a^2+b^2+c^2=4+abc.$ Chứng minh rằng $$(a^2+2)(b^2+2)(c^2+2)\ge 9+6(ab+bc+ca).$$
 


Nguyen Van Huyen
Ho Chi Minh City University Of Transport

#63
Nguyenhuyen_AG

Nguyenhuyen_AG

    Trung úy

  • Thành viên nổi bật 2016
  • 945 Bài viết

Bài 31 (Iran TST). Cho bốn số thực dương $a,b,c,d$ thỏa mãn điều kiện $\frac{1}{a+1}+\frac{1}{b+1}+\frac{1}{c+1}+\frac{1}{d+1}=2.$ Chứng minh rằng
$$\sqrt{\frac{a^2+1}{2}} +\sqrt{\frac{b^2+1}{2}}+\sqrt{\frac{c^2+1}{2}}+\sqrt{\frac{d^2+1}{2}} +8 \geqslant 3(\sqrt{a}+\sqrt{b}+\sqrt{c}+\sqrt{d}).$$

Bài 32. (Kyiv Mathematical Festival)

1. Cho ba số thực không âm $a,b,c$ thỏa mãn điều kiện $ab+bc+ca=2.$ Chứng minh rằng \[\frac{ab}{c+1}+\frac{bc}{a+1}+\frac{ca}{b+1}+2(a+b+c) \geqslant 6.\]
2. Cho ba số thực không âm $a,b,c$ thỏa mãn điều kiện $ab+bc+ca=3.$ Chứng minh rằng \[\frac{ab}{c+1}+\frac{bc}{a+1}+\frac{ca}{b+1} \geqslant \frac{3}{2}.\]


 


Bài viết đã được chỉnh sửa nội dung bởi Nguyenhuyen_AG: 10-05-2016 - 20:11

Nguyen Van Huyen
Ho Chi Minh City University Of Transport

#64
canhhoang30011999

canhhoang30011999

    Thiếu úy

  • Thành viên
  • 634 Bài viết

Bài 31 (Iran TST). Cho bốn số thực dương $a,b,c,d$ thỏa mãn điều kiện $\frac{1}{a+1}+\frac{1}{b+1}+\frac{1}{c+1}+\frac{1}{d+1}=2.$ Chứng minh rằng
$$\sqrt{\frac{a^2+1}{2}} +\sqrt{\frac{b^2+1}{2}}+\sqrt{\frac{c^2+1}{2}}+\sqrt{\frac{d^2+1}{2}} +8 \geqslant 3(\sqrt{a}+\sqrt{b}+\sqrt{c}+\sqrt{d}).$$

Bài 32. (Kyiv Mathematical Festival)

1. Cho ba số thực không âm $a,b,c$ thỏa mãn điều kiện $ab+bc+ca=2.$ Chứng minh rằng \[\frac{ab}{c+1}+\frac{bc}{a+1}+\frac{ca}{b+1}+2(a+b+c) \geqslant 6.\]
2. Cho ba số thực không âm $a,b,c$ thỏa mãn điều kiện $ab+bc+ca=3.$ Chứng minh rằng \[\frac{ab}{c+1}+\frac{bc}{a+1}+\frac{ca}{b+1} \geqslant \frac{3}{2}.\]


 

31 

ta sẽ cm $\sqrt{\frac{a^{2}+1}{2}}-3\sqrt{a}\geq \frac{4}{a+1}-4$

$\Leftrightarrow \frac{(a+1)(a-1)^{2}}{2(\sqrt{\frac{a^{2}+1}{2}}+\sqrt{a})}-2\sqrt{a}(\sqrt{a}-1)^{2}\geq 0$

do $\sqrt{\frac{a^{2}+1}{2}}+\sqrt{a}\leq a+1$

nên ta cần cm $(\sqrt{a}-1)^{2}(\frac{(\sqrt{a}+1)^{2}}{2}-2\sqrt{a})\geq 0$(luôn đúng)

thiết lập các bất đẳng thức tương tự ta có đpcm

p\s có ai làm được bài Nga chưa?


Bài viết đã được chỉnh sửa nội dung bởi canhhoang30011999: 10-05-2016 - 22:13


#65
NTA1907

NTA1907

    Thượng úy

  • Thành viên
  • 1014 Bài viết

Bài 32. (Kyiv Mathematical Festival)

1. Cho ba số thực không âm $a,b,c$ thỏa mãn điều kiện $ab+bc+ca=2.$ Chứng minh rằng \[\frac{ab}{c+1}+\frac{bc}{a+1}+\frac{ca}{b+1}+2(a+b+c) \geqslant 6.\]

2. Cho ba số thực không âm $a,b,c$ thỏa mãn điều kiện $ab+bc+ca=3.$ Chứng minh rằng \[\frac{ab}{c+1}+\frac{bc}{a+1}+\frac{ca}{b+1} \geqslant \frac{3}{2}.\]

2. Áp dụng Cauchy-Schwarz ta có:

$\sum \frac{(ab)^{2}}{abc+ab}\geq \frac{(ab+bc+ca)^{2}}{3abc+ab+bc+ca}=\frac{3}{abc+1}\geq \frac{3}{2}$(vì $abc\leq 1$ theo AM-GM)

Dấu = xảy ra khi $a=b=c=1$

1. Tương tự câu 2 ta chứng minh được:

$\sum \frac{ab}{c+1}\geq 6-2\sqrt{6}$

Mà $2(a+b+c)\geq 2\sqrt{3(ab+bc+ca)}=2\sqrt{6}$

Cộng 2 bất đẳng thức trên ta có đpcm


Bài viết đã được chỉnh sửa nội dung bởi NTA1907: 10-05-2016 - 22:41

Vũ trụ không có biên trong không gian, không có bắt đầu và kết thúc trong thời gian và chẳng có việc gì cho đấng sáng thế phải làm ở đây cả.

 


#66
the unknown

the unknown

    Thượng sĩ

  • Thành viên
  • 208 Bài viết

Bài 30 (EMMO). Cho ba số thực $a,b,c$ thỏa mãn điều kiện $a^2+b^2+c^2=4+abc.$ Chứng minh rằng $$(a^2+2)(b^2+2)(c^2+2)\ge 9+6(ab+bc+ca).$$
 

Áp dụng bất đẳng thức AM-GM ta có $a^2+b^2+c^2=4+abc\geq 3\sqrt[3]{a^2b^2c^2}$

Đặt $\sqrt[3]{abc}=t\Rightarrow t^3+4\geq 3t^2\Leftrightarrow (t+1)(t-2)^2\geq 0\Leftrightarrow t\geq -1\Rightarrow abc\geq -1\Rightarrow a^2+b^2+c^2\geq 3$.

Ta sẽ chứng minh $3(a+b+c)^2\leq (a^2+2)(b^2+2)(c^2+2)$.

Ta có $(a+b+c)^2=(a.1+\sqrt{2}.\frac{b+c}{\sqrt{2}})^2\leq (a^2+2)(1+\frac{(b+c)^2}{2})$.

Do đó ta chỉ cần chứng minh $(b^2+2)(c^2+2)\geq 3(1+\frac{(b+c)^2}{2})\Leftrightarrow (bc-1)^2+\frac{(b-c)^2}{2}\geq 0$ ( luôn đúng)

Vậy $(a^2+2)(b^2+2)(c^2+2)\geq 3(a+b+c)^2=3(a^2+b^2+c^2)+6(ab+bc+ca)\geq 9+6(ab+bc+ca)$ ( do $a^2+b^2+c^2\geq 3$).

Vậy ta có đpcm.


$\texttt{If you don't know where you are going, any road will get you there}$


#67
Nguyenhuyen_AG

Nguyenhuyen_AG

    Trung úy

  • Thành viên nổi bật 2016
  • 945 Bài viết

Mình tổng hợp lại một số bài chưa có lời giải, mọi người cùng suy nghĩ thử nhé :)

 

Bài 20 (China Mathematical Olympiad). Cho $a_1,a_2,\ldots, a_{31},\,b_1,b_2, \ldots, b_{31}$ là các số nguyên dương thỏa mãn


$$a_1< a_2<\cdots< a_{31}\leqslant 2015,\; b_1< b_2<\cdots<b_{31} \leqslant 2015,$$

$$a_1+a_2+\cdots+a_{31}=b_1+b_2+\cdots+b_{31}.$$
Tìm giá trị lớn nhất của $S=|a_1-b_1|+|a_2-b_2|+\cdots+|a_{31}-b_{31}|.$

 


Bài 21 (European Girls' Mathematical Olympiad). Cho $n$ là số nguyên dương lẻ và $x_1,x_2,...,x_n$ là các số thực không âm.

Chứng minh rằng: $\min_{1\le i\le n} (x_i^2+x_{i+1}^2)\le\max_{1\le j\le n} (2x_jx_{j+1}).$

 

Bài 23 (Romania JBMO TST 2016). Với $m,n$ là hai số tự nhiên và ba số thực $x,y,z$ thuộc $[0,1].$ Chứng minh rằng
\[0 \leqslant x^{m+n}+y^{m+n}+z^{m+n}-x^my^n-y^mz^n-z^mx^n \leqslant 1.\]
Đẳng thức xảy ra khi nào ?

Bài 27 (Iran Second Round). Cho ba số thực $0<a \leqslant b \leqslant c.$ Chứng minh rằng
$$\frac{(c-a)^2}{6c}\leq \frac{a+b+c}{3}-\frac{3}{\frac{1}{a}+\frac{1}{b}+\frac{1}{c}}.$$

Bài 28 (Russia). Cho bốn số thực dương $a, b, c, d$ thỏa mãn điều kiện $a+b+c+d=3.$ Chứng minh rằng $$\frac{1}{a^2}+\frac{1}{b^2}+\frac{1}{c^2}+\frac{1}{d^2}\le\frac{1}{a^2b^2c^2d^2},$$


$$\frac{1}{a^3}+\frac{1}{b^3}+\frac{1}{c^3}+\frac{1}{d^3}\le\frac{1}{a^3b^3c^3d^3}.$$

Bài 29 (EMMO). Tìm số thực dương $\lambda$ nhỏ nhất sao cho bất đẳng thức
$$\frac{\sqrt[3]{a^3+b^3}}{a^2+b^2}+\frac{\sqrt[3]{b^3+c^3}}{b^2+c^2}+\frac{\sqrt[3]{c^3+a^3}}{c^2+a^2}\le \lambda\left(1+\frac{1}{ab+bc+ca}\right),$$

luôn đúng với mọi số dương $a,b,c$ thỏa mãn điều kiện $a+b+c=1.$

 


Nguyen Van Huyen
Ho Chi Minh City University Of Transport

#68
Nguyen Minh Hai

Nguyen Minh Hai

    Thiếu úy

  • Thành viên
  • 666 Bài viết

 

Mình tổng hợp lại một số bài chưa có lời giải, mọi người cùng suy nghĩ thử nhé :)

 

 

Bài 23 (Romania JBMO TST 2016). Với $m,n$ là hai số tự nhiên và ba số thực $x,y,z$ thuộc $[0,1].$ Chứng minh rằng

\[0 \leqslant x^{m+n}+y^{m+n}+z^{m+n}-x^my^n-y^mz^n-z^mx^n \leqslant 1.\]
Đẳng thức xảy ra khi nào ?

 

$(a)$      Chứng minh $ x^{m+n}+y^{m+n}+z^{m+n} \geqslant x^my^n+y^mz^n+z^mx^n$

Gọi $(a,b,c)$ hoán vị của $(x,y,z)$ sao cho $a \geqslant b \geqslant c$

 

Khi đó ta $a^m \geqslant b^m \geqslant c^m$ $a^n \geqslant b^n \geqslant c^n$.

 

Do đó theo BĐT Hoán vị ta

$$a^ma^n+b^mb^n+c^mc^n \geqslant a^mb^n+b^mc^n+c^ma^n$$

$$a^ma^n+b^mb^n+c^mc^n \geqslant a^mc^n+b^ma^n+c^ma^n$$

ta

$$a^ma^n+b^mb^n+c^mc^n=x^{m+n}+y^{m+n}+z^{m+n}$$

$$x^my^n+y^mz^n+z^mx^n \in \lbrace a^mb^n+b^mc^n+c^ma^n , a^mc^n+b^ma^n+c^ma^n \rbrace$$

Do đó vế trái của BĐT được chứng minh. Xảy ra đẳng thức khi $x=y=z$

$(b)$         Chứng minh

$$x^my^n+y^mz^n+z^mx^n+1 \geqslant x^{m+n}+y^{m+n}+z^{m+n}$$

 

Giả sử $x=max\lbrace x,y,z \rbrace$ thì BĐT được viết lại thành

 

$$(1-x^{m+n})+y^n(x^m-y^m)+z^m(x^n-z^n)+y^mz^n \geqslant 0$$

 

BĐT cuối luôn đúng do $x=max\lbrace x,y,z \rbrace$ $x,y,z \in [0;1]$.

 

Vậy vế phải BĐT được chứng minh. Xảy ra đẳng thức khi $x=1,y=z=0$ các hoán vị. $\square$


Bài viết đã được chỉnh sửa nội dung bởi Nguyen Minh Hai: 15-05-2016 - 21:25


#69
Nguyen Minh Hai

Nguyen Minh Hai

    Thiếu úy

  • Thành viên
  • 666 Bài viết

Một cách tiếp cận khác cho Bài 30

Bài 30 (EMMO). Cho ba số thực $a,b,c$ thỏa mãn điều kiện $a^2+b^2+c^2=4+abc.$ Chứng minh rằng $$(a^2+2)(b^2+2)(c^2+2)\ge 9+6(ab+bc+ca).$$
 

Đặt $t = \sqrt[3]{abc}$ thì từ giả thiết bài toán ta

$4+abc = a^{2}+b^{2}+c^{2} \geqslant 3\sqrt[3]{a^{2}b^{2}c^{2}}$

$\Leftrightarrow 4+t^3 \geqslant 3t^2  \Leftrightarrow  (t+1)(t+2)^2 \geqslant 0$

 

Từ đó suy ra $abc \geqslant -1$, kết hợp với giả thiết suy ra $a^{2}+b^{2}+c^{2} \geqslant 3$.

 

Ta BĐT quen thuộc sau $(a^{2}+2)(b^{2}+2)(c^{2}+2) \geqslant 3(a+b+c)^{2}   (1)$

 

Thật vậy, theo BĐT $Cauchy - Schwarz$ thì ta

 

$$(a+b+c)^{2} \leqslant (a^{2}+2)\left[1+\frac{(b+c)^2}{2}\right]$$

 

Do đó để chứng minh $(1)$ ta chỉ cần chứng minh

$$(b^{2}+2)(c^{2}+2) \geqslant 3\left[1+\frac{(b+c)^2}{2}\right]$$

 

$$\Leftrightarrow \frac{1}{2}(b-c)^2+(bc-1)^2 \geqslant 0$$

 

Do đó $(1)$ được chứng minh.

 

 Áp dụng $(1)$ thì ta quy bài toán về chứng minh

 

$$(a+b+c)^{2} \geqslant 3+2(ab+bc+ca)$$

 

$$\Leftrightarrow a^{2}+b^{2}+c^{2} \geqslant 3$$ 

 

BĐT này luôn đúng. Do đó bài toán được chứng minh 

 Xảy ra đẳng thức khi $a=b=c=-1$. $\square$


Bài viết đã được chỉnh sửa nội dung bởi Nguyen Minh Hai: 16-05-2016 - 00:47


#70
Nguyen Minh Hai

Nguyen Minh Hai

    Thiếu úy

  • Thành viên
  • 666 Bài viết

 

Mình tổng hợp lại một số bài chưa có lời giải, mọi người cùng suy nghĩ thử nhé :)

 

 

Bài 27 (Iran Second Round). Cho ba số thực $0<a \leqslant b \leqslant c.$ Chứng minh rằng
$$\frac{(c-a)^2}{6c}\leq \frac{a+b+c}{3}-\frac{3}{\frac{1}{a}+\frac{1}{b}+\frac{1}{c}}.$$

 

Hình gửi kèm

  • iran second round.jpg


#71
Nguyen Minh Hai

Nguyen Minh Hai

    Thiếu úy

  • Thành viên
  • 666 Bài viết

 

Mình tổng hợp lại một số bài chưa có lời giải, mọi người cùng suy nghĩ thử nhé :)

 

Bài 28 (Russia). Cho bốn số thực dương $a, b, c, d$ thỏa mãn điều kiện $a+b+c+d=3.$ Chứng minh rằng $$\frac{1}{a^2}+\frac{1}{b^2}+\frac{1}{c^2}+\frac{1}{d^2}\le\frac{1}{a^2b^2c^2d^2},$$


$$\frac{1}{a^3}+\frac{1}{b^3}+\frac{1}{c^3}+\frac{1}{d^3}\le\frac{1}{a^3b^3c^3d^3}.$$

 

 

Lời giải 

 $(a)$       Giả sử $a \geqslant b \geqslant c \geqslant d$, BĐT cần chứng minh tương đương với

$a^{2}b^{2}c^{2}+a^{2}b^{2}d^{2}+a^{2}c^{2}d^{2}+b^{2}c^{2}d^{2} \leqslant 1$

Ta đưa BĐT về dạng đồng bậc

$a^{2}b^{2}c^{2}+a^{2}b^{2}d^{2}+a^{2}c^{2}d^{2}+b^{2}c^{2}d^{2} \leqslant \frac{1}{3^6}(a+b+c)^6$

Do $a \geqslant b \geqslant c \geqslant d$ nên ta

\[\begin{aligned} a^{2}b^{2}c^{2}+a^{2}b^{2}d^{2}+a^{2}c^{2}d^{2}+b^{2}c^{2}d^{2} &\leqslant a^{2}b^{2}c^{2}+a^{2}b^{2}d^{2}+a^{2}b^{2}cd+a^{2}b^{2}cd \\ &= a^{2}b^{2}(c+d)^{2} \\ &\leqslant \frac{1}{3^{6}}(a+b+c+d)^{6} \\ &=1\end{aligned}\]

Bài toán được chứng minh. $\square$ 

 $(b)$       Giả sử $a \geqslant b \geqslant c \geqslant d$, BĐT cần chứng minh tương đương với

$a^{3}b^{3}c^{3}+a^{3}b^{3}d^{3}+a^{3}c^{3}d^{3}+b^{3}c^{3}d^{3} \leqslant 1$

Ta đưa BĐT về dạng đồng bậc

$a^{3}b^{3}c^{3}+a^{3}b^{3}d^{3}+a^{3}c^{3}d^{3}+b^{3}c^{3}d^{3} \leqslant \frac{1}{3^9}(a+b+c)^9$

Theo BĐT $AM - GM$ ta

$ \frac{1}{3^9}(a+b+c+d)^9 \geqslant \frac{1}{3^9}\left(3\sqrt[3]{ab(c+d)} \right)^9 = a^3b^3(c+d)^3 $

Do đó ta chỉ cần chứng minh $a^{3}b^{3}c^{3}+a^{3}b^{3}d^{3}+a^{3}c^{3}d^{3}+b^{3}c^{3}d^{3} \leqslant a^3b^3(c+d)^3$ $\Leftrightarrow c^3d^3(a^3+b^3) \leqslant 3a^3b^3cd(c+d)$ $\Leftrightarrow c^2d^2(a^3+b^3) \leqslant 3a^3b^3(c+d)$

BĐT này luôn đúng do $a \geqslant b \geqslant c \geqslant d$

Bài toán được chứng minh. $\square$



#72
Nguyenhuyen_AG

Nguyenhuyen_AG

    Trung úy

  • Thành viên nổi bật 2016
  • 945 Bài viết

Bài 33 (Saudi Arabia TST). Cho ba số thực dương $a,b,c$ thỏa mãn điều kiện $a+b+c=3.$ Chứng minh rằng \[\frac{1}{a^2+b^2+c^2}+\frac{1}{\sqrt{abc}} \geqslant \frac{4}{3}.\]


Bài viết đã được chỉnh sửa nội dung bởi Nguyenhuyen_AG: 22-05-2016 - 11:57

Nguyen Van Huyen
Ho Chi Minh City University Of Transport

#73
Ngockhanh99k48

Ngockhanh99k48

    Trung sĩ

  • Thành viên
  • 127 Bài viết

Bài 33 (Saudi Arabia TST). Cho ba số thực dương $a,b,c$ thỏa mãn điều kiện $a+b+c=3.$ Chứng minh rằng \[\frac{1}{a^2+b^2+c^2}+\frac{1}{\sqrt{abc}} \geqslant \frac{4}{3}.\]

.
Đặt $t=ab+bc+ca$, với $t \in (0;3]$.
Sử dụng liên tiếp AM-GM ta có $3^6 = [a^2+b^2+c^2+2(ab+bc+ca)]^3 \geq 27(a^2+b^2+c^2)(ab+bc+ca)^2$ $\Leftrightarrow$ $\frac{1}{a^2+b^2+c^2} \geq \frac{t^2}{27}$. Mặt khác $3 \geq t \geq 3 \sqrt[3]{a^2b^2c^2}$. Do đó $\frac{1}{\sqrt{abc}} \geq \sqrt{\dfrac{27}{t^3}}$. Từ đó ta có $\frac{1}{a^2+b^2+c^2} + \frac{1}{\sqrt{abc}} \geq \frac{t^2}{27} + \sqrt{\frac{27}{t^3}} = \frac{t^2}{81}.3 + \frac{1}{9}\sqrt{\frac{27}{t^3}}.4 + \dfrac{5}{9}\sqrt{\frac{27}{t^3}} \geq 7\sqrt[7]{(\frac{t^2}{81})^3.(\dfrac{1}{9}\sqrt{\frac{27}{t^3}})^4} + \frac{5}{9} = \frac{4}{3}$. Dấu bằng xảy ra khi $a=b=c=1$.

Bài viết đã được chỉnh sửa nội dung bởi Ngockhanh99k48: 22-05-2016 - 13:01


#74
Nguyenhuyen_AG

Nguyenhuyen_AG

    Trung úy

  • Thành viên nổi bật 2016
  • 945 Bài viết

Bài 34 (Serbia Junior TST). Cho $a,b,c$ là ba số thực dương. Chứng minh rằng $$\frac{2a}{\sqrt{3a+b}}+\frac{2b}{\sqrt{3b+c}}+\frac{2c}{\sqrt{3c+a}}\leq \sqrt{3(a+b+c)}.$$
 


Nguyen Van Huyen
Ho Chi Minh City University Of Transport

#75
Ankh

Ankh

    Hạ sĩ

  • Thành viên
  • 85 Bài viết

Bài 34 (Serbia Junior TST). Cho $a,b,c$ là ba số thực dương. Chứng minh rằng $$\frac{2a}{\sqrt{3a+b}}+\frac{2b}{\sqrt{3b+c}}+\frac{2c}{\sqrt{3c+a}}\leq \sqrt{3(a+b+c)}.$$
 

 Áp dụng bất đẳng thức Cauchy-Schwarz ta có $\sum \dfrac{2a}{\sqrt{3a+b}}\leq 2\sqrt{(a+b+c)\sum \dfrac{a}{3a+b}}$

 Nên ta chỉ cần chứng minh $\sum \dfrac{a}{3a+b}\leq \dfrac{3}{4}\Leftrightarrow \sum \dfrac{b}{3a+b}\geq \dfrac{3}{4}$

 Áp dụng bất đẳng thức Cauchy-Schwarz ta có $\sum \dfrac{b}{3a+b}=\sum \dfrac{b^2}{3ab+b^2}\geq \dfrac{(a+b+c)^2}{(a+b+c)^2+ab+bc+ca}\geq \dfrac{3}{4}$

 Dấu "=" xảy ra khi $a=b=c$



#76
Nguyenhuyen_AG

Nguyenhuyen_AG

    Trung úy

  • Thành viên nổi bật 2016
  • 945 Bài viết

Bài 35 (Turkey JBMO TST). Chứng minh rằng
\[(x^4+y)(y^4+z)(z^4+x) \geqslant (x+y^2)(y+z^2)(z+x^2),\]
trong đó $x, y, z$ là ba số thực dương thỏa mãn điều kiện $xyz \geqslant 1.$


Nguyen Van Huyen
Ho Chi Minh City University Of Transport

#77
minhrongcon2000

minhrongcon2000

    Thượng sĩ

  • Thành viên
  • 213 Bài viết

Bài 35 (Turkey JBMO TST). Chứng minh rằng
\[(x^4+y)(y^4+z)(z^4+x) \geqslant (x+y^2)(y+z^2)(z+x^2),\]
trong đó $x, y, z$ là ba số thực dương thỏa mãn điều kiện $xyz \geqslant 1.$

Ta có bất đẳng thức sau: $(x^{4}+y)(\frac{z^{2}}{y}+1)\geqslant (x^{2}+z)^{2}$ (chứng minh dễ dàng bằng phép tương đương) $\Leftrightarrow (x^{4}+y)(z^{2}+y) \geqslant y(x^{2}+z)^{2}$.

Chứng minh tương tự, ta có

$(y^{4}+z)(x^{2}+z) \geqslant z(y^{2}+x)^{2}$
$(z^{4}+x)(y^{2}+x) \geqslant x(z^{2}+y)^{2}$

Nhân vế theo vế, ta có
$VT \geqslant xyz*VP \geqslant VP$.
Ta có đpcm

Bài viết đã được chỉnh sửa nội dung bởi minhrongcon2000: 25-05-2016 - 10:01

$\lim_{x \to \infty } Love =+\infty$


#78
Nguyenhuyen_AG

Nguyenhuyen_AG

    Trung úy

  • Thành viên nổi bật 2016
  • 945 Bài viết

Bài 36 (Austrian MO). Cho ba số thực $a,\,b,\,c \geqslant -1$ thỏa mãn điều kiện $a^3+b^3+c^3=1.$ Chứng minh rằng \[a+b+c+a^2+b^2+c^2 \leqslant 4.\]

Đẳng thức xảy ra khi nào ?


Nguyen Van Huyen
Ho Chi Minh City University Of Transport

#79
fatcat12345

fatcat12345

    Binh nhất

  • Banned
  • 46 Bài viết

Bài 36 (Austrian MO). Cho ba số thực $a,\,b,\,c \geqslant -1$ thỏa mãn điều kiện $a^3+b^3+c^3=1.$ Chứng minh rằng \[a+b+c+a^2+b^2+c^2 \leqslant 4.\]

Đẳng thức xảy ra khi nào ?

Sử dụng bất đẳng thức $a^2+a-a^3-1\leq 0\Leftrightarrow (a+1)(a-1)^2\geq 0$

Xây dựng các bất đẳng thức tượng tự rồi cộng lại ta có đpcm.

Đẳng thức xảy ra khi và chỉ khi $(a,b,c)=(1,1,-1)$ và các hoán vị.


Bài viết đã được chỉnh sửa nội dung bởi fatcat12345: 27-05-2016 - 13:35


#80
fatcat12345

fatcat12345

    Binh nhất

  • Banned
  • 46 Bài viết

Bài 29 (EMMO). Tìm số thực dương $\lambda$ nhỏ nhất sao cho bất đẳng thức
$$\frac{\sqrt[3]{a^3+b^3}}{a^2+b^2}+\frac{\sqrt[3]{b^3+c^3}}{b^2+c^2}+\frac{\sqrt[3]{c^3+a^3}}{c^2+a^2}\le \lambda\left(1+\frac{1}{ab+bc+ca}\right),$$

luôn đúng với mọi số dương $a,b,c$ thỏa mãn điều kiện $a+b+c=1.$

Cho $a=b=c=\frac{1}{3}$ suy ra $\lambda \geq \frac{9\sqrt[3]{2}}{8}.$

Ta cần chứng minh

$$\sum_{cyc}\frac{\sqrt[3]{a^3+b^3}}{a^2+b^2}\leq \frac{9\sqrt[3]{2}}{8}(1+\frac{1}{\sum_{cyc}ab})$$

Sử dụng bất đẳng thức sau $\sqrt[3]{\frac{a^3+b^3}{2}}\leq \frac{a^2+b^2}{a+b}$

Nên chỉ cần chứng minh

$$\sum_{cyc}\frac{1}{a+b}\leq \frac{9}{8}(1+\frac{1}{\sum_{cyc}ab})$$

Bất đẳng thức này đúng, thật vậy

$$\sum_{cyc}\frac{1}{a+b}=\frac{\sum_{cyc}a^2+3\sum_{cyc}ab}{(a+b)(b+c)(c+a)}\leq \frac{9}{8}\frac{\sum_{cyc}a^2+3\sum_{cyc}ab}{(\sum_{cyc}ab)(\sum_{cyc}a)}=\frac{9}{8}(1+\frac{1}{\sum_{cyc}ab})\blacksquare $$






2 người đang xem chủ đề

0 thành viên, 2 khách, 0 thành viên ẩn danh